Wednesday 14 August 2019

momentum - Klein Gordon Field Quantization: why this is the correct way to express the field?


I'm reading a book in QFT and the first thing tackled is the quantization of the Klein Gordon Field. The classical Klein Gordon field satisfies the partial differential equation


$$(\partial^\mu\partial_\mu+m^2)\phi=0 \Longrightarrow (\partial_t^2-\nabla^2+m^2)\phi=0.$$


Taking the Fourier transform we get


$$(\partial_t^2+\omega_p^2)\hat{\phi}=0,$$


where now $\omega_p^2 = p^2+m^2$. In other words $\hat{\phi}(p,t)$ satisfies the simple harmonic oscilator equation of motion for each fixed $p$. In this case we have


$$\phi(x,t)=\int d^3p \dfrac{1}{(2\pi)^3}\hat{\phi}(p,t)e^{ix\cdot p}.$$


That is fine, and is all classical. Now, we want to quantize the field. As the book explains, quantizing the field means promoting $\phi(\mathbf{x},t)$ to an operator, such that


$$[\phi(\mathbf{x}),\phi(\mathbf{y})]=[\pi(\mathbf{x}),\pi(\mathbf{y})]=0,$$


$$[\phi(\mathbf{x}),\pi(\mathbf{y})]=(2\pi)^3\delta(\mathbf{y}-\mathbf{x}).$$



in the same way as we do with position $X$ and momentum $P$ in quantum mechanics.


Now, to do this the author uses the ladder operators from the quantum harmonic oscilator. In that case if $X$ and $P$ are position and momentum, the ladder oerators satisfies


$$X=\dfrac{1}{\sqrt{2\omega}}(a+a^\dagger), \quad P=-i\sqrt{\dfrac{\omega}{2}}(a-a^\dagger).$$


The author by analogy with this, then says that


$$\phi(x)=\int d^3p \dfrac{1}{(2\pi)^3}\dfrac{1}{\sqrt{2\omega_p}}(a_pe^{ix\cdot p}+a_p^\dagger e^{-ix\cdot p})$$ $$\pi(x)=\int d^3p \dfrac{1}{(2\pi)^3}(-i)\sqrt{\dfrac{\omega_p}{2}}(a_pe^{ix\cdot p}-a_p^\dagger e^{-ix\cdot p})$$


Now this is not at all clear to me. My main issues are:



  1. First what leads to this expansion of the quantized field $\phi$? I mean, my guess is that the author considered $\hat{\phi}(p)$ behaves as the coordinate of a harmonic oscilator, so that we can write $\hat{\phi}(p)$ in terms of ladder operators $a_p$ and $a_p\dagger$. However if that is done we would get


$$\phi(x)=\int d^3p \dfrac{1}{(2\pi)^3}\dfrac{1}{\sqrt{2\omega_p}}(a_p+a_p^\dagger)e^{ix\cdot p}$$



This would be the analogy in my opinion, where we set $\hat{\phi}(p)$ as the position of the harmonic oscilator. Why do we get $(a_p e^{ix\cdot p}+a_p^\dagger e^{-ix\cdot p})$ instead?



  1. If this analogy is being carried, why $\hat{\pi}(p)$ would be the momentum of the harmonic oscilator? I see no reason for this directly by the differential equation.




No comments:

Post a Comment

Understanding Stagnation point in pitot fluid

What is stagnation point in fluid mechanics. At the open end of the pitot tube the velocity of the fluid becomes zero.But that should result...